Questions tagged [inequalities]

for questions involving inequalities, upper and lower bounds.

Filter by
Sorted by
Tagged with
3 votes
2 answers
551 views

Upper bound for complex integral

I am interested in obtaining a good upper bound for the absolute value of the following integral $$ \left| \int_{0}^{\pi/3} e^{-itn} \left( 1-e^{it} \right)^{k} dt \right|, $$ when $n>k>0$ are ...
user512026's user avatar
1 vote
2 answers
120 views

Approximation of the Gaussian Mills' ratio

Let $R(t) = \frac{1 - \Phi(t)}{\varphi(t)}$ where $\Phi, \varphi$ represent the CDF and PDF of the standard Normal distribution, respectively. I am interested in approximations of the function for $t &...
Drew Brady's user avatar
1 vote
1 answer
156 views

Inequalities involving entropy: quantum discord and mutual information

My question is inspired by the following paper of Olivier and Żurek but for this question to be self-contained I will recall all the necessary definitions: for a quantum state $\rho$ we define the ...
truebaran's user avatar
  • 9,150
1 vote
1 answer
123 views

Optimal constant comparing $f(1/2)$ and $\|f\|_2$ when $f$ is $t$-Hölder?

Suppose that $f \colon [0, 1] \to \mathbb{R}$ is $k$ times continuously differentiable and Holder in the sense that for some $t = k + \beta$, where $\beta \in (0, 1]$ and $k$ is a nonnegative integer ...
Drew Brady's user avatar
3 votes
2 answers
266 views

Is the passage in argument of existence solution of PDE correct?

The passage below is from a fixed point argument in Wang's paper enter link description here pg 566. At the end of the $I_1$ calculation, it somehow makes the following estimate $$C\|\phi\|_X (e^{-|x|^...
Ilovemath's user avatar
  • 585
2 votes
2 answers
165 views

Relating function value to $L^2$ norm in Holder space

Suppose that $f \colon [0, 1] \to \mathbb{R}$ is $k$ times continuously differentiable and Holder in the sense that for some $t = k + \beta$, where $\beta \in (0, 1]$ and $k$ is a nonnegative integer ...
Drew Brady's user avatar
4 votes
1 answer
394 views

Inequality of inclusion-exclusion term

This question was initially posted on math.stackexchange.com but did not receive any answers for half a week. While analyzing the properties of an algorithm I am working on (I'm a computer scientist), ...
Tobias's user avatar
  • 45
3 votes
1 answer
165 views

What is the minimum and the maximum perimeter of a triangle with area $x$ that can be inscribed in a circle?

This question was posted in MSE but is still open hence posting in MO. The area of the largest triangle that can be inscribed in a circle of raidus $1$ is $\displaystyle \frac{3 \sqrt{3}}{4}$ for a ...
Nilotpal Kanti Sinha's user avatar
1 vote
1 answer
129 views

Kolmogorov inequality for Bernoulli random variables

This question is also asked on math stackexchange. The question is about one inequality which shows in Kolmogorov's paper (inequality (3.1)) but is not proved. The inequality says that, if we assume $...
Greenhand's user avatar
5 votes
0 answers
160 views

Is there a sharper Golden–Thompson inequality?

For any two Hermitian matrices $A$ and $B$, the Golden–Thompson inequality $$\mathrm{Tr} (e^A e^B) \geq \mathrm{Tr} \, e^{A + B}$$ holds, and it is known to be a strict inequality whenever $[A, B] \...
Karen H.'s user avatar
3 votes
1 answer
141 views

Derive distributional inequalities from pointwise estimates

My question is how to prove the following claim: Suppose that $E$ is an algebraic set in $\mathbb{R}^n (n\ge3)$ with dimension $\le n-2$, and $u$ is locally Lipschitz continuous on $\mathbb{R}^n$. If ...
William's user avatar
  • 33
16 votes
1 answer
750 views

Find a special integer coefficients polynomial which takes small absolute value on [0,4]

The question is easy to state: Is there a non-constant $f\in\mathbb{Z}[x]$ such that for all $x\in [0,4]$, we have $|f(x)|\leq 1$? I do not know where to find a useful reference for it. I did a few ...
Yanlong Hao's user avatar
2 votes
0 answers
313 views

Minimizing $P=\frac{\sqrt{5a+8bc}}{8a+5bc}+\frac{\sqrt{5b+8ca}}{8b+5ca}+\frac{\sqrt{5c+8ab}}{8c+5ab}.$ [closed]

Olympiad inequality. Let $a,b,c\ge 0: ab+bc+ca=1.$ Find the minimal value $P$ of $$f:=\frac{\sqrt{5a+8bc}}{8a+5bc}+\frac{\sqrt{5b+8ca}}{8b+5ca}+\frac{\sqrt{5c+8ab}}{8c+5ab}.$$ Note: Often Stack ...
Dragon boy's user avatar
1 vote
1 answer
152 views

Ratio of the constants of the Marcinkiewicz–Zygmund inequality for p=1

The Marcinkiewicz–Zygmund inequality states that $$ {\displaystyle A_{p}E\left(\left(\sum _{i=1}^{n}\left\vert X_{i}\right\vert ^{2}\right)_{}^{p/2}\right)\leq E\left(\left\vert \sum _{i=1}^{n}X_{i}\...
allsisyphus's user avatar
0 votes
0 answers
46 views

Weighted version of the Gagliardo Nirenberg inequality

I'm searching for a weighted Gagliardo-Nirenberg inequality similar as in https://arxiv.org/pdf/1307.1363.pdf where the weight is a power of the last component. Is there an inequality of the form $$ \...
user99432's user avatar
  • 173
0 votes
1 answer
229 views

A question about the prime counting function

I was playing around with the prime counting function and came across something that seemed correct to me, maybe it's already been proven but I don't know so I decided to ask here. maybe a stupid ...
Egehan Eren's user avatar
5 votes
1 answer
300 views

Recover unknown vector through shifted argmax queries

$\DeclareMathOperator*{\argmax}{arg\,max}$ I am interested in finding an efficient algorithm for the following problem: Let $x \in [0,1]^n$ be some vector, with $x_n = 1$. We want to recover $x$, ...
Florian Tramèr's user avatar
4 votes
1 answer
201 views

Erdős–Rényi random graphs — reproducing 2 inequalities

In Erdős and Renyi's 1959 paper On random graphs I , I'm trying to reproduce, starting from Eq.\eqref{1} in their paper, the two inequalities that appear in Eq.\eqref{2}. Eq.\eqref{1} is: $$ P \le \...
RickB88's user avatar
  • 43
2 votes
0 answers
164 views

How to prove this weighted sum inequality with non-increasing sequences?

Problem I have two non-increasing sequences, $X = (x_1, x_3, x_5, \ldots, x_{n-1})$ and $Y = (y_1, y_3, y_5, \ldots, y_{n -1})$, $n$ is an even integer. I want to prove this inequality: $$ \sum_{i=1}^{...
birdlpy's user avatar
  • 29
6 votes
1 answer
349 views

Maximizing a sum minus its maximal summand

This is a followup to a question that appeared on m.SE: Maximize $\displaystyle f(\pi)=\left(\sum_{i=1}^{n}{i\pi_i}\right)-\max_{1\le i\le n}{(i\pi_i)}$ over permutations $\pi\in S_n$. The problem ...
Alexander Burstein's user avatar
2 votes
1 answer
112 views

Explicit upper and lower bounds for a certain support function

Let $a_1 \geq a_2 \geq \cdots \geq a_n \geq 0$ be a sequence of nonincreasing nonnegative real numbers. Define the set, for $t > 1$, $$ B_t = \Big\{b \in \mathbb{R}^n : b_i \geq 0, \sum_i b_i^2 \...
Drew Brady's user avatar
4 votes
2 answers
464 views

How to get this inequality in Santambrogio's book about optimal transport?

Let $\hat{\varrho}, \tilde{\varrho}$ be probability density functions on $\mathbb R^d$ where $\tilde{\varrho} \in L^{\infty} (\mathbb R^d)$. For $\varepsilon \in [0, 1]$, we define $\varrho_{\...
Akira's user avatar
  • 851
0 votes
1 answer
67 views

Inequality for extremal values of product of Hermitian matrices

I am looking for a reference to verify the following inequality, where $X$ and $Y$ are Hermitian positive semidefinite matrices: $$ \lambda_n(X^{1/2}YX^{1/2}) = \lambda_n(XY) \leq \lambda_n(X)\...
turtlesandwich's user avatar
6 votes
1 answer
762 views

A Poincaré-like inequality

Is it true that for some real $K>0$ and all real $u\in C_0^\infty((0,1))$ we have $$\int_0^1 (u'(x)^2+u(x)^2)\,dx\,\int_0^1 u(x)^2\,dx \le K\Big(\int_0^1 x\,u'(x)^2\,dx\Big)^2\text{ ?}$$
Iosif Pinelis's user avatar
51 votes
24 answers
10k views

Most elementary proof showing that exponential growth wins against polynomial growth

This question is motivated by teaching : I would like to see a completely elementary proof showing for example that for all natural integers $k$ we have eventually $2^n>n^k$. All proofs I know rely ...
1 vote
0 answers
59 views

A possible upper bound for a function that satisfies a singular integral inequality

I am currently working on an analysis problem in fractional calculus and after some work I have encountered the following inequality: $$ |v(s)|~\leq \epsilon+\beta \int_{0}^{1}|s-x|^{\alpha-1}\left( |...
Takieddine Zeghida's user avatar
0 votes
1 answer
160 views

For which $p>p^*$ does the inequality $\cos^2(−π/4+π/p)>1/2+π/p^2$​ hold?

I have the inequality $\cos^2(−\frac{\pi}{4}+\frac{\pi}{p})>\frac{1}{2}+\frac{π}{p^2}$​​ and have found that it holds for $p>p^*$, where $p^*$ is some positive number (around ~2.8). I'm looking ...
blockchain_dietmar's user avatar
0 votes
0 answers
59 views

Friedrich's second inequality for functions with zero average

Friedrich's second inequality (or Maxwell Estimates or Gaffney’s inequality in the literature) is referred as follows: for all $\mathbf{u} \in H^1(\Omega)^2$ satisfying either $\mathbf{n} \cdot \...
Ryan Li's user avatar
  • 21
6 votes
2 answers
235 views

Does "perpendicular phase incoherence" satisfy the triangle inequality?

I asked this question at https://math.stackexchange.com/q/4783968/222867, but even after a 200-point bounty, no solution was provided, only some thoughts regarding possible directions. So I'm now ...
Julian Newman's user avatar
0 votes
2 answers
213 views

Bounds tighter than the additive Chernoff

Additive Chernoff Suppose $X_1, \ldots, X_n$ are i.i.d. random variables, taking values in $\{0,1\}$. Let $p=\mathrm{E}\left[X_i\right]$ and $\varepsilon>0$. \begin{gather*} \operatorname{Pr}\left(\...
Dotman's user avatar
  • 105
8 votes
3 answers
556 views

Jensen-like inequality for random matrix: $\Bbb E[\det X^2]\ge\det\Bbb E[X^2]$

Let $X\in M_n(\Bbb R)$ be a random matrix with iid elements following a continuous distribution. What are the necessary and sufficient conditions for $$\Bbb E[\det X^2]\ge\det\Bbb E[X^2]$$ to hold? Is ...
ə̷̶̸͇̘̜́̍͗̂̄︣͟'s user avatar
0 votes
1 answer
96 views

Is it possible obtain an identity of the type $\dfrac{|x-y|}{t} + \dfrac{|y-z|}{s} \approx a(t,s) | y - b(t,x,s,z)| + c(t,s)|x-z|$?

using the Euclidean inner product, and by completing squares it is possible to prove that $$\dfrac{|x-y|^2}{t} + \dfrac{|y-z|^2}{s} = \dfrac{t+s}{ts} \left| y - \dfrac{sx+tz}{s+t} \right|^2 + \dfrac{|...
Ilovemath's user avatar
  • 585
1 vote
1 answer
118 views

estimate involving Gaussian data

Let $x\in \mathbb{R}^{d}$, $d\geq 1$ and $p>\frac{2}{d}$. For any $R>0$ and any $0<s<\frac{dp}{2}-1$ \begin{align} &\int_{\mathbb{R}}\int_{|x|>R}\left(\frac{1}{1+t^2} \right)^{\frac{...
Julian Bejarano's user avatar
8 votes
0 answers
329 views

Bounding a sum of reciprocals of square-free integers

(Cross-posted from MSE, as the question did not get any clear answer) Fix positive integers $k$ and $n$. Let $N_1,\dots,N_r$ be all the integers less than or equal to $n$ that are squarefree and have ...
Juan Moreno's user avatar
0 votes
1 answer
108 views

Integral inner product with exponential function

Suppose on some unknown interval $[0, I]$ we have non-negative functions $f, g : [0, I] \rightarrow \mathbb{R}^{\geq 0}$. If we know that \begin{aligned} \int_0^I f & = c \\ \int_0^I e^f & = e^...
Lewwwer's user avatar
  • 129
2 votes
1 answer
361 views

On norm of the Sobolev space $H^2(\Omega)$, $\Omega \subset \mathbb{R}^n; n \geq 2$

Let the Sobolev space $H^2(\Omega)$ be defined with the norm $\|u\|_{H^2(\Omega)}=\Big(\sum_{|\alpha|\leq 2})\|D^{\alpha}u\|^2_{L^2(\Omega)}\Big)^\frac{1}{2}$. I have found in several research ...
Akundu's user avatar
  • 41
3 votes
0 answers
209 views

Do these cousins of permanents satisfy the following inequality?

Let $H$ denote an $n$ by $n$ hermitian positive semidefinite matrix. Let $G$ and $K$ be two subgroups of the symmetric group $\Sigma_n$. Define $$ f_{G, K}(H) = \sum_{(\sigma, \tau) \in G \times K} \...
Malkoun's user avatar
  • 5,011
3 votes
1 answer
236 views

Sub-Gaussian random variables and convex ordering

Suppose that $X$ is a $1$-sub-Gaussian real-valued random variable, i.e. for all $t \in \mathbf{R}$, it holds that $\log \mathbf{E} \exp \left( t X \right) \leqslant \frac{1}{2} t^2 $. Does there ...
πr8's user avatar
  • 706
1 vote
1 answer
52 views

Possibility of bounding one functional by another functional (under certain constraints)

Suppose that we consider a class of $L^2(\mathbb{R}_+)$ functions $h$ such that $h$ can be expressed as a difference of two cumulative distribution functions $F$ and $G$ (whose corresponding densities ...
Fei Cao's user avatar
  • 700
1 vote
0 answers
73 views

Help with understanding a proof of existence of solutions

In El Fatini and Boukanjime "Stochastic analysis of a two delayed epidemic model incorporating Lévy processes with a general non-linear transmission" paper, can someone give a detailed ...
Leo's user avatar
  • 121
1 vote
0 answers
46 views

Can we bound the squared Gaussian curvature of genus three triply periodic minimal surfaces?

Assume that $\mathcal{M}$ is a balanced triply periodic minimal surface of genus 3, embedded in a flat torus $T^3=\mathbb{R}^3/\Lambda$ for a lattice $\Lambda$ with volume 1. I want to understand the ...
Matthias Himmelmann's user avatar
11 votes
1 answer
3k views

Understanding the application of two inequalities?

I am reading the paper "The long-time behaviour of a stochastic SIR epidemic model with distributed delay and multidimensional Levy jumps" by Driss Kiouach and Yassine Sabbar. I have two ...
Math's user avatar
  • 185
2 votes
1 answer
117 views

Simplified upper bounds for moment-generating function of symmetrised random variable

Let $X$ be a nonnegative random variable such that $\mathbf{E} \left[ \exp X \right] < \infty$. For $\theta \leqslant 1$, an appropriate application of Jensen's inequality, yields that \begin{align}...
πr8's user avatar
  • 706
0 votes
1 answer
219 views

Equivalence between the $L^2$ norm and the $L^2$ norm of Laplace transform

It is well-known that the Laplace transform, defined by $$\mathcal{L} \colon f(x) \in L^2(\mathbb{R}_+) \to \hat{f}(\xi) \in L^2(\mathbb{R}_+)$$ via $$\hat{f}(\xi) = \int_{\mathbb{R}_+} f(x)\,\mathrm{...
Fei Cao's user avatar
  • 700
12 votes
4 answers
910 views

Understanding the condition $\frac{1}{p} + \frac{1}{q} = 1$ in the estimate $xy \le \frac{1}{p}x^p + \frac{1}{q}y^q$

I just read a proof of Holder's inequality in measure theory, which boils down to the following inequality: $$xy \le \frac{1}{p}x^p + \frac{1}{q}y^q$$ where $x,y\ge 0$ and $\frac{1}{p} + \frac{1}{q} = ...
stupid_question_bot's user avatar
1 vote
1 answer
107 views

$L^1$ error between indicator function and smoothed out version

For a large parameter $r>0$, consider the indicator function $1_{[-r,r]}$ and its convolution with the (normalized) Gaussian $\frac{1}{\sqrt{\pi}}e^{-x^2}$, that is, $$f_r(x) = \frac{1}{\sqrt{\pi}}\...
Staki42's user avatar
  • 151
2 votes
1 answer
186 views

A question about equivalence of weighted Sobolev space norm in S. Benzoni-Gavage and D. Serre's book

This question may not be at the research level, but it has really bothered me for a long time. The following space is used for handling initial boundary value problem for first order hyperbolic ...
vent de la paix's user avatar
13 votes
2 answers
2k views

One specific inequality

How to prove this inequality $$\left(a+\frac{1}{2} \left(a b-\sqrt{a^2-1} \sqrt{b^2-1}\right)\right)^{3/4}-\frac{\sqrt{3} \cos\left[\frac{3 (\pi -t)}{4}\right]}{2 \left(\frac{1}{2}+b\right)^{1/4}}-(a+\...
MathArt's user avatar
  • 333
1 vote
0 answers
116 views

Inequality concerning the imaginary parts of a recurrent sequence, Laplacian eigenvectors

Let $u=(u_1,\dots,x_n)\in\mathbb{C}^n$ be a sequence that satisfies the cyclic recurrence $$ \lambda+1 =a_{i-1}\frac {u_{i-1}}{u_i} + (1-a_{i+1})\frac{ u_{i+1} }{u_i } $$ with $a_i \in (0,1)$ and $\...
Artemy's user avatar
  • 650
4 votes
2 answers
760 views

Is there any bilinear Poincaré/Sobolev inequality?

Is the following, I call it bilinear Poincaré inequality, true? Let $\Omega$ be an open bounded set in $\mathbf R^n\DeclareMathOperator{\dL}{d\!}$. There exists $C > 0$ such that for any $u, v \in ...
Hao Yu's user avatar
  • 185

1
2
3 4 5
34